Đến nội dung

nhungvienkimcuong nội dung

Có 204 mục bởi nhungvienkimcuong (Tìm giới hạn từ 08-05-2020)



Sắp theo                Sắp xếp  

#734612 [TOPIC] HÌNH HỌC PHẲNG

Đã gửi bởi nhungvienkimcuong on 24-08-2022 - 20:41 trong Hình học

Giờ nhìn hình các bạn làm khiếp thật, bỏ hình lâu quá nên giờ chẳng thể làm bài nào của các bạn cả. Mạn phép các bạn mình đăng một bài, lục lại vở cũ thấy ngày xưa mình có làm bài toán sau.

 

Bài 39: Cho tam giác $ABC$ (góc $B$ lớn hơn góc $C$) và $D$ là điểm trên $AC$ sao cho $\angle ABD =\angle C$. Đặt $I$ là tâm nội tiếp của tam giác $ABC$ và đường tròn ngoại tiếp tam giác $CDI$ cắt $AI$ ở $E$ khác $I$. Đường thẳng đi qua $E$ và song song với $AB$ cắt $BD$ ở $P$. Đặt $J$ là tâm nội tiếp tam giác $ABD$, $A’$ là điểm đối xứng với $A$ qua $I$. Goị $Q$ là giao của $JP$ và $A’C$. Chứng minh rằng $QJ=QA’$.




#734353 $x_1+x_2+x_3+…+x_7=31$

Đã gửi bởi nhungvienkimcuong on 13-08-2022 - 09:14 trong Tổ hợp - Xác suất và thống kê - Số phức

Và vô cùng đặc biệt
$$\sum_{k=1}^n f(n,k) = f(2n,n)$$
Ví dụ: $f(14,9)=f(14,7+2)=f(12,7)=f(12,6+1)=f(11,6)=f(11,5+1)=f(10,5)=f(9,4)+1=f(8,3)+2=5+2=7$
——
Chứng minh rằng:

$$f(n,k)=\sum_{i=1}^k f(n-k,i)$$

Những kết quả như này có rất nhiều cách chứng minh: biểu đồ Ferrer, quy nạp bằng định nghĩa của $f(n,k)$ hoặc hệ thức truy hồi. Sau đây là một cách sử dụng hàm sinh.

 

Đầu tiên thì kí hiệu hệ số của $x^n$ trong $F(x)$ là $[x^n]F(x)$. Tiếp theo là nêu lại một số kết quả khá quen thuộc

Kết quả 1.

$$f(n,k)=[x^n]\frac{x^k}{\prod_{j=1}^{k}(1-x^j)}.\tag{1}$$

Kết quả 2.

$$\sum_{i=0}^k\left(x^i\prod_{j=i+1}^k(1-x^j) \right )=1.\tag{2}$$

Chứng minh hai kết quả này không khó khắn lắm, kết quả 1 thì khai triển vế phải, còn kết quả 2 thì quy nạp. Cuối cùng ta chứng minh hệ thức đề bài như sau

$$\begin{align*}  \sum_{i=1}^kf(n-k,i)&\overset{(1)}{=}\sum_{i=1}^k[x^{n-k}]\frac{x^i}{\prod_{j=1}^i(1-x^j)}\\&=[x^{n-k}]\sum_{i=1}^k\frac{x^i}{\prod_{j=1}^i(1-x^j)}\\&=[x^{n-k}]\sum_{i=1}^k\frac{x^i\prod_{j=i+1}^k(1-x^j)}{\prod_{j=1}^k(1-x^j)}\\&\overset{(2)}{=}[x^{n-k}]\frac{1}{\prod_{j=1}^k(1-x^j)}\\&=[x^n]\frac{x^k}{\prod_{j=1}^k(1-x^j)}\\&\overset{(1)}{=}f(n,k).\end{align*}$$

Với $n:=2n,k:=n$ thì $f(n)=\sum_{k=1}^nf(n,k)=f(2n,n)$.




#734286 $x_1+x_2+x_3+…+x_7=31$

Đã gửi bởi nhungvienkimcuong on 09-08-2022 - 21:36 trong Tổ hợp - Xác suất và thống kê - Số phức

2. Chứng minh rằng:
$f(n,3)=\left\lfloor \frac{n^2+3}{12}\right\rfloor$

Việc tính $f(n,3)$ cũng chính là đếm số nghiệm nguyên dương của phương trình $x+y+z=n$ với $x\ge y\ge z$ (Cái này đã được thầy Thanh và anh Kool LL xử lí rất chi tiết ở đây  :luoi:)

 

Ngoài ra thì có một cách khác như này: một đẳng thức rất quan trọng trong phân hoạch là $f(n,k)=f(n-1,k-1)+f(n-k,k)$. Với $k=3$ ta có

$$f(n,3)=f(n-1,2)+f(n-3,3).$$

Dễ thấy $f(n,2)=\left \lfloor \frac{n}{2} \right \rfloor$, tới đây quy nạp là thu được điều cần chứng minh. 




#734348 $x_1+x_2+x_3+…+x_7=31$

Đã gửi bởi nhungvienkimcuong on 12-08-2022 - 20:17 trong Tổ hợp - Xác suất và thống kê - Số phức

Một đề mới thi yêu cầu tính liên quan tới $f(n,2)$ và $f(n,3)$. Chắc đáp án của BTC kì thi cũng trùng với cách nào đó đã được nêu ở đây thôi nhỉ  :lol:

299020533_5792701130764080_1514290455805928667_n.jpg




#734183 [TOPIC] PTH $\mathbb{R} \rightarrow \mathbb{R}$ 2022

Đã gửi bởi nhungvienkimcuong on 03-08-2022 - 09:50 trong Phương trình hàm

Em nghĩ chúng ta nên phản chứng dựa vào

$$\begin{CD}f\left ( x \right )= x @>{{\it into}\;{\it it}}>> f\left ( 3y+ 1 \right )= 3f\left ( y \right )+ 1\\ @AA{{\it as}\;{\it hypothesis}}A @VV{{\it after}\;{\it substituted}}V\\ f\left ( 3f\left ( x \right )+ x \right )= 3f\left ( x \right )+ x @<{\it And}<< f\left ( 3f\left ( y \right )+ 1 \right )= 3f\left ( y \right )+ 1\end{CD}$$

Khá giống với việc chứng minh $f\left ( f\left ( f\left ( x \right ) \right ) \right )= x\Rightarrow f\left ( x \right )= x$ như ở đây.

Cái link em đưa có giả thiết $f$ tăng ngặt còn bài của em thì chưa có.

Ngoài ra sao có thể suy ra $f(3f(y)+1)=3f(y)+1$ được nhỉ? Nếu có cái này thì xong bài rồi, vì hàm đã cho có tính chất $f(f(x))=x$ sau khi có được $f(0)=0$.




#734194 [TOPIC] PTH $\mathbb{R} \rightarrow \mathbb{R}$ 2022

Đã gửi bởi nhungvienkimcuong on 04-08-2022 - 08:43 trong Phương trình hàm

Bài 3. Tìm tất cả các hàm $f:\;\mathbb{R}\rightarrow\mathbb{R}$ thỏa mãn

$$\forall_{\left ( x, y \right )\in\mathbb{R}^{2}}\;f\left ( 2f\left ( xy \right )+ xf\left ( y \right )+ f\left ( x \right ) \right )= 3yf\left ( x \right )+ x$$

Bạn nào giúp mình chứng minh $f(1)=1$ với, ban đầu mình có làm nhưng giờ xem lại thì bị sai mất  :wacko: . Sau đây là lời giải khi đã có $f(1)=1$.

 

Dễ thấy $f$ không thể là hàm hằng, thay $x:=0$ vào giả thiết thu được $f(0)=0$. Thay $y:=0$ vào giả thiết có được $f(f(x))=x$, do đó ta viết lại giả thiết như sau

$$f(3yf(x)+x)=2f(xy)+xf(y)+f(x).\tag{$\ast$}$$

Với $f(1)=1$ lần lượt thay $y:=1$ và $x:=1$ vào $(\ast)$ ta có

$$\begin{align}f(3f(x)+x)&=3f(x)+x\\ f(3y+1)&=3f(y)+1 \end{align}$$

Đặt tập hợp điểm bất động là $\mathcal{U}=\Big\{u\in \mathbb{R}:f(u)=u\Big\}$. Từ đây ta kí hiệu $u$ là phần tử bất kì của $\mathcal{U}$.

$\bullet$ Chứng minh $\frac{u}{3}\in \mathcal{U}$ với mọi $u\in\mathcal{U}$.

$\bullet$ Chứng minh

\begin{equation}f(3x)=4x-3f\left(\frac{x}{3}\right),\quad \forall x\in \mathbb{R}.\end{equation}

Thay $x:=\frac{1}{3}$ vào $(\ast)$ ta có

\begin{equation} f\left(y+\frac{1}{3}\right)=2f\left(\frac{y}{3}\right)+\frac{f(y)}{3}+\frac{1}{3}\end{equation}

Thay $x:=3$ vào $(\ast)$ ta có

$$\begin{align*} f(9y+3)&=2f(3y)+3f(y)+3\\\overset{(3)}{\implies}4(3y+1)-3f\left ( y+\frac{1}{3} \right )&=2f(3y)+3f(y)+3\\\overset{(4)}{\implies}4(3y+1)-3\left ( 2f\left(\frac{y}{3}\right)+\frac{f(y)}{3}+\frac{1}{3} \right )&=2f(3y)+3f(y)+3\end{align*}$$

Tương đương với

$$12y=2\left(3f\left(\frac{y}{3}\right)+f(3y)\right)+4f(y)\overset{(3)}{=}8y+4f(y).$$

Từ đây ta có được $f(y)=y$.




#734071 [TOPIC] PTH $\mathbb{R} \rightarrow \mathbb{R}$ 2022

Đã gửi bởi nhungvienkimcuong on 22-07-2022 - 15:09 trong Phương trình hàm

Bài 2: Tìm tất cả các hàm số $f:\mathbb R\to\mathbb R$ thoả mãn $$f(x - f(y)) = 4f(x) + 3x + f(y),\forall x,y\in\mathbb R$$

Thay $x:=\frac{x-f(y)}{3}$ vào giả thiết ta có

$$f\left(\frac{x-f(y)}{3}-f(y)\right)=4f\left(\frac{x-f(y)}{3}\right)+x.$$

Do vậy ta có nhận xét: Với mọi $x$ thì luôn tồn tại $u,v$ sao cho $x=f(u)-4f(v).\qquad (1)$

Thay $x:=f(y)$ vào giả thiết ta có

$$4f(f(y))+4f(y)=f(0).\qquad (2)$$

Thay $x:=f(x)-f(y)$ vào giả thiết ta có

\begin{align*}f(f(x)-f(y))&=4f(f(x))+3f(x)+f(y)\\&\overset{(2)}{=} f(0)-f(x)+f(y).\end{align*}

Hoàn toàn tương tự, lần lượt thay $x$ bởi $f(x)-2f(y),f(x)-3f(y)$ ta thu được

$$f(f(x)-4f(y))=-f(x)+4f(y)+64f(0).$$

Kết hợp với $(1)$ dẫn tới $f(x)\equiv -x+64c$, phần còn lại là thay vào giả thiết và tìm $c$ thôi.

 

P/s: Ngày xưa mình ưng làm dạng này lắm  :lol: (ai muốn luyện tập thêm cho dạng này thì ở đây).




#734180 [TOPIC] PTH $\mathbb{R} \rightarrow \mathbb{R}$ 2022

Đã gửi bởi nhungvienkimcuong on 03-08-2022 - 09:00 trong Phương trình hàm

$f\left ( 1 \right )= 1$ đồng thời

$$f\left ( 3y+ 1 \right )= 3f\left ( y \right )+ 1\quad{\it And}\quad f\left ( 3f\left ( x \right )+ x \right )= 3f\left ( x \right )+ x$$

Bài này mình cũng làm được tới đây, sau đó thì như thế nào nhỉ?




#731306 [TOPIC] Mỗi ngày một bài toán IMO

Đã gửi bởi nhungvienkimcuong on 25-10-2021 - 15:29 trong Các dạng toán khác

Bài 8: [IMO 1987] Chứng minh rằng không tồn tại hàm $f$ nào từ tập hợp các số nguyên không âm vào chính nó thoả mãn $f(f(n))=n+1987$ với mọi $n$.

 

Bài này ta có thể tổng quát lên như sau: Tồn tại hàm số $f\colon \mathbb{N}\to \mathbb{N}$ sao cho $\underbrace{f(f(\cdots f}_{a\ \text{lần}}(n)\cdots))=n+b$ với mọi $n\in \mathbb{N}$ khi và chỉ khi $a\mid b$

 

Ở đây mình chỉ làm theo đề gốc, bài toán tổng quát các bạn dành cho các bạn  :D

Giả sử tồn tại hàm $f$ thỏa đề. Từ giả thiết dễ thấy $f(n+1987)=f(n)+1987\ \color{Red}{(1)}$.

Tiếp đến ta làm việc trong $ \mathbb{Z}_{1987}$, có được

$$f(f(x))=x,\quad \forall x\in \mathbb{Z}_{1987}$$

Xây dựng đồ thị có hướng $[V,E]$ trong đó tập đỉnh chính là các phần tử của tập hợp $\mathbb{Z}_{1987}$, có cạnh $[v_1,v_2]$ nếu $v_2=f(v_1)$. Với cách xây dựng này thì ta thấy mỗi đỉnh của đồ thị thuộc $1$-chu trình hoặc $2$-chu trình. Vì $1987$ là số lẻ nên tồn tại điểm bất điểm bất động $m\in \mathbb{Z}_{1987}$, nghĩa là $f(m)=m$.

Quay trở lại làm việc trong $\mathbb{Z}$. ta có được $f(m)=m+1987k$ với $k$ là số nguyên nào đó. Do vậy

\[f(f(m))=f(m+1987k)\overset{\color{Red}{(1)}}{=}f(m)+1987k=m+1987\cdot 2k\]

Mặt khác theo giả thiết thì $f(f(m))=m+1987$, suy ra $2k=1$ (vô lí).




#737712 [HOT HOT HOT] ĐÃ CÓ KẾT QUẢ THI HSG QUỐC GIA 2023

Đã gửi bởi nhungvienkimcuong on 14-03-2023 - 04:56 trong Thi HSG Quốc gia và Quốc tế

Bạn thủ khoa này không lạ mặt gì với diễn đàn cả, chúc mừng @Hoang72




#734550 Chứng minh $a+11b+13c=m^2$ luôn có nghiệm

Đã gửi bởi nhungvienkimcuong on 21-08-2022 - 17:58 trong Số học

Em lăn tăn vụ một trong hai số $b$ hoặc $c$ có thể bằng $0$ nên nghĩ hơi nhiều rồi. “Tăng” điều kiện của $n$ lên là có nghiệm nguyên dương thôi, phải không nào!

Đúng rồi thầy ạ  :icon6: . Vậy lời giải ở trên của thầy là chuẩn rồi.

Ngoài ra với cách làm của thầy thì hoàn toàn có thể tìm được một điều kiện khá ổn cho bài tổng quát anh Hân nêu ở trên nữa.




#734538 Chứng minh $a+11b+13c=m^2$ luôn có nghiệm

Đã gửi bởi nhungvienkimcuong on 20-08-2022 - 18:43 trong Số học

Hay $5b+6c=\frac{m^2-n}{2}\;\;\;(*)$
Theo định lý Sylvester thì điều kiện cần để $(*)$ có nghiệm là $\frac{m^2-n}{2}>5\times 6=30$

Định lí Sylvester chỉ ra có nghiệm tự nhiên, nghĩa là dù thỏa mãn điều kiện cần thì vẫn có thể xảy ra trường hợp $b=0$ hoặc $c=0$. Em cũng bị lúng túng chỗ này nên khúc chặn bên trên làm cho $N$ hơi lớn  :icon6:




#734548 Chứng minh $a+11b+13c=m^2$ luôn có nghiệm

Đã gửi bởi nhungvienkimcuong on 21-08-2022 - 15:25 trong Số học

Sau đây mình hiệu chỉnh lại lời giải của thầy Thanh, ngoài ra chặn được tốt hơn lời giải phía trên của mình nữa  :icon6:

Với $n\ge 40$ cho trước, ta sẽ chứng minh tồn tại bộ ba số nguyên dương $(b,c,m)$ thỏa mãn

$$5b+6c=\frac{m^2-n}{2}\tag{$\blacklozenge$}$$

và $b+c<n$.

 

Vì $n\ge 40$ nên có bất đẳng thức sau $\sqrt{n+40}+12\le \sqrt{11n}$. Do vậy có ít nhất $12$ số nguyên nằm trong đoạn $\Big[\sqrt{n+40},\sqrt{11n}\Big].\qquad$ $(1)$

Sau đây là bảng số dư khi chia $m^2$ cho $3$ và $5$

$$\begin{array}{c|ccccccccccccccc} m&0 & 1&2&3&4&5&6&7&8&9&10&11&12&13&14\\ \hline m^2\equiv\ \ ?\pmod{3} & 0 & 1&1 & 0 & 1&1 & 0 & 1&1 & 0 & 1&1 & 0 & 1&1\\ \hline m^2\equiv\ \ ?\pmod{5} & 0 & 1&4 & 4 & 1&0 &1 & 4&4 & 1 & 0&1 & 4 & 4&1\end{array}$$

Dựa vào bảng này thì ta thấy rằng với số $n$ cho trước thì trong $12$ số nguyên liên tiếp luôn tồn tại $m$ sao cho

$$\left\{\begin{gathered}m^2-n\equiv 0\pmod{2}\\ m^2-n\not\equiv 0\pmod{3}\\m^2-n\not\equiv 0\pmod{5}\end{gathered}\right.\qquad \color{red}{(2)}$$

Từ $(1)$ và $(2)$ suy ra tồn tại số nguyên $m$ thỏa mãn hệ đồng dư $(2)$ và $n+40\le m^2\le 11n$. Phần còn lại thì tương tự ở trên của mình. 

 

Chú thích: Tại sao phải là $12$ số nguyên liên tiếp?




#734530 Chứng minh $a+11b+13c=m^2$ luôn có nghiệm

Đã gửi bởi nhungvienkimcuong on 20-08-2022 - 15:23 trong Số học

Bài này em làm chưa triệt để, nhưng tạm thời chứng minh được với $n$ đủ lớn thì thỏa đề.

Problem
Chứng minh rằng: Với mọi số nguyên dương $n\ge 3$, ta luôn tách được $n$ thành tổng của ba số nguyên dương $a,b,c$ sao cho $a+11b+13c$ là một số chính phương.

Sau đây em sẽ chứng minh với mọi $n\ge N$ thì thỏa đề, trong đó $N$ (khá lớn) như thế nào sẽ xuất hiện ở phần sau.

Gọi $k$ là số nguyên dương thỏa mãn $(k-1)^2<n\le k^2$ và $r\in \{0,1,\dots,29\}$ thỏa mãn hệ sau

$$\left\{\begin{gathered}(3k+r)^2-n\equiv 0\pmod{2}\\ (3k+r)^2-n\not\equiv 0\pmod{3}\\(3k+r)^2-n\not\equiv 0\pmod{5}\end{gathered}\right.$$

Vì $n=a+b+c$ nên $a+11b+13c=n+10b+12c$. Tiếp đến ta sẽ chứng minh rằng tồn tại $b,c$ nguyên dương sao cho $b+c<n$ và

$$n+10b+12c=(3k+r)^2\iff 5b+6c=\frac{(3k+r)^2-n}{2}.\tag{$\blacklozenge$}$$

$\bullet$ Chứng minh tồn tại $b,c\in \mathbb{N}^*$.

$\bullet$ Chứng minh $b+c<n$.

 

Chú thích (Định lí Sylvester): Cho hai số nguyên dương $a,b$ sao cho $\gcd(a,b)=1$, khi đó với mọi $n\ge ab-a-b+1$ thì phương trình $ax+by=n$ luôn có nghiệm tự nhiên $x,y$.

 

 

 




#733869 $0< | a+b \sqrt{2} + c \sqrt{3}| <...

Đã gửi bởi nhungvienkimcuong on 03-07-2022 - 06:47 trong Các dạng toán khác

Nếu mình hiểu đúng thì mệnh đề này sẽ suy ra là tập $\{a+b\sqrt{q}\}$ có tính trù mật nhỉ?

Phải là ngược lại mới đúng á anh  :lol: . Vì tập $\left\{a+b\sqrt{q}:a,b\in \mathbb{Z}\right\}$ trù mật trong $\mathbb{R}$ nếu với hai số dương $\epsilon_1,\epsilon_2$ bất kì ($\epsilon_1<\epsilon_2$) thì tồn tại $a,b$ sao cho $\epsilon_1<\left|a+b\sqrt{q}\right|<\epsilon_2$.




#742704 Đề thi HSG 9 THPT chuyên Amsterdam

Đã gửi bởi nhungvienkimcuong on 25-12-2023 - 15:01 trong Tài liệu - Đề thi

Lớp 9 học giải phương trình đa thức rồi à :D

Các bạn lớp 9 hiện tại học nhiều ghê anh ạ  :wacko: .

 

Bài V.

1) Cho $2024$ số nguyên dương $x_1,x_2,\dots,x_{2024}$ được viết thành một hàng ngang theo thứ tự đó, thỏa mãn $x_1=1$ và với mỗi $k\in\{,1,2,\dots,2024\}$, tổng của $k$ số liên tiếp bất kì trong hàng chia hết cho $x_k$. Chứng minh rằng $x_{2024}\le 2^{1012}-1$.

Với mỗi số nguyên $n$ đặt $X_n=x_1+x_2+\dots+x_n$, từ giả thiết dễ thấy

\[x_n\mid X_{n-1},\qquad x_{n+1}\equiv 1\pmod{x_n}.\tag{$\ast$}\]

Trước tiên ta sẽ nháp để xem các giá trị của dãy số như thế nào. Dễ thấy $x_2=1$ và $x_3\in \{1,2\}$, với hai trường hợp của $x_3$ thì ta sẽ tính các giá trị khác với mong muốn mỗi giá trị $x_n$ đạt giá trị lớn nhất, đồng thời thỏa mãn điều kiện $(\ast)$.

\[\begin{array}{c|cccccccccc}n& 1& 2& 3& 4& 5& 6& 7& 8& 9& 10\\\hline x_n& 1& 1& \color{red}{2}& 1& 1& 2& 1& 3& 4& 1 \\ X_n& 1& 2& 4& 5& 6& 8& 9& 12& 16&  \end{array}\qquad \begin{array}{c|cccccccccc}n& 1& 2& 3& 4& 5& 6& 7& 8& 9& 10\\\hline x_n& 1& 1& \color{red}{1}& 3& 1& 7& 1& 15& 1& 31 \\ X_n& 1& 2& 3& 6& 7& 14& 15& 30& 31&  \end{array}\]

Từ đây ta thấy rằng các giá trị của $x_{2n}$ lớn hơn khi $x_3=1$ và cũng phù hợp với yêu cầu chứng minh của đề bài :icon6: . Cũng dựa vào bảng giá trị thì nghĩ đến việc quy nạp đồng thời

\[X_{2n}\le 2^{n+1}-2\qquad\text{và}\qquad x_{2n}\le 2^n-1.\]

Spoiler



#733856 $0< | a+b \sqrt{2} + c \sqrt{3}| <...

Đã gửi bởi nhungvienkimcuong on 02-07-2022 - 07:21 trong Các dạng toán khác

Chứng minh rằng với mọi $\epsilon >0$ Tồn tại các số nguyên $a;b;c$ sao cho :

 

$0< | a+b \sqrt{2} + c \sqrt{3}| < \epsilon$

Ta thấy rằng tồn tại vô hạn $n$ nguyên dương thỏa mãn $2n^2-\left \lfloor n\sqrt{2} \right \rfloor^2=1$ (xem thêm ở đây). Khi đó

$$n\sqrt{2}-\left \lfloor n\sqrt{2} \right \rfloor=\frac{1}{n\sqrt{2}+\left \lfloor n\sqrt{2} \right \rfloor}<\frac{1}{2n\sqrt{2}-1}.$$

Vì tồn tại vô hạn $n$ nên hoàn toàn có thể chọn $n$ sao cho $\frac{1}{2n\sqrt{2}-1}<\epsilon$, dẫn tới 

$$0<\left|\left \lfloor n\sqrt{2} \right \rfloor-n\sqrt{2}+0\cdot\sqrt{3}\right|<\epsilon.$$

Vậy với $a=\left \lfloor n\sqrt{2} \right \rfloor, b=-n$ và $c=0$ thì thỏa đề.




#742673 Đề thi HSG Toán 9, tỉnh Thái Bình năm học 2023-2024

Đã gửi bởi nhungvienkimcuong on 24-12-2023 - 10:30 trong Tài liệu - Đề thi

Bài 7. Cho a, b là các số nguyên thỏa mãn $a,b\neq -1$ và $\frac{(a-1)(a+1)^{2}+(b-1)(b+1)^{2}}{a+b+ab+1}$ là số nguyên.

            Chứng minh rằng $a^{2023}b^{2024}-a$ chia hết cho $a^{2}+a$.

Thấy rằng

\[\frac{(a-1)(a+1)^{2}+(b-1)(b+1)^{2}}{a+b+ab+1}=\frac{b^2-1}{a+1}+\frac{a^2-1}{b+1}.\]

Đặt $\frac{b^2-1}{a+1}=\frac{x}{y}$ và $\frac{a^2-1}{b+1}=\frac{z}{t}$ với $x,y,z,t$ là các số tự nhiên sao cho ƯCLN$(x,y)=$ƯCLN$(z,t)=1$. Theo giả thiết thì 

\[\frac{xt+yz}{yt}=\frac{x}{y}+\frac{z}{t}\in\mathbb{Z}\implies yt\mid xt+yz\implies y\mid xt\implies y\mid t.\]

Ngoài ra 

$$\frac{x}{y}\cdot \frac{z}{t}=(a-1)(b-1)\in\mathbb{Z}\implies yt\mid xz\implies y\mid z.$$

Mà ƯCLN$(z,t)=1$ nên $y=1$. Như vậy $\frac{b^2-1}{a+1}$ là số nguyên, nghĩa là $a+1\mid b^2-1$. Phần còn lại thì biến đổi

$$a^{2023}b^{2024}-a=ab^{2024}(a^{2022}-1)+a(b^{2024}-1).$$

Vì $a+1$ là ước của cả $a^{2022}-1$ lẫn $b^{2024}-1$ nên ta có điều cần chứng minh.

 

Ghi chú. Bài này dựa trên Đề thi học sinh giỏi Quốc gia năm 2007.




#741951 $\lim_{n\to \infty }\sqrt[n]{1+cos(2n...

Đã gửi bởi nhungvienkimcuong on 31-10-2023 - 21:09 trong Giải tích

Tính giới hạn sau:

$\lim_{n\to \infty }\sqrt[n]{1+cos(2n)}$

Nhìn lướt qua thì cảm giác đây là bài toán bình thường cho phổ thông, đến lúc đặt bút thì chịu  :icon6: . Tìm trên mạng thấy xuất hiện ở đây.




#742203 CMR: $a+b+c \geq ab + bc + ac$.

Đã gửi bởi nhungvienkimcuong on 24-11-2023 - 07:56 trong Bất đẳng thức và cực trị

Bài này có hướng giải theo cách lớp 9 không ạ
Cho $a, b, c$ là các số thực không âm thoả mãn:

$ab + bc + ac + abc\leq 4$. CMR: $$ab+bc+ac \leq a+b+c.$$

Theo nguyên lí Đi-rich-lê thì trong ba số $a-1,b-1,c-1$ phải có hai số cùng dấu; giả sử là $a-1$ và $b-1$. Khi đó

\[c(a-1)(b-1)\ge 0\implies c\ge ac+bc-abc\implies a+b+c\ge a+b-abc+ac+bc.\]

Như vậy ta cần chứng minh

\[a+b-abc+ac+bc\ge ab+bc+ca\iff a+b\ge ab(c+1).\]

Mặt khác theo giả thiết thì $c\le \frac{4-ab}{a+b+ab}$ (ở đây đang xét $a^2+b^2\neq 0$), do đó ta cần chứng tỏ

\[a+b\ge ab\left(\frac{4-ab}{a+b+ab}+1\right)\iff \frac{(a-b)^2}{a+b+ab}\ge 0.\]




#736645 CM $\exists g:\mathbb R\mapsto \mathbb R$ thỏa...

Đã gửi bởi nhungvienkimcuong on 06-01-2023 - 08:44 trong Phương trình - Hệ phương trình - Bất phương trình

Với $\varepsilon >0$ cho trước. Hàm $f:\mathbb R\mapsto \mathbb R$ thỏa mãn

$$\left|f(x+y)-f(x-y)-2f(y)\right|\le \varepsilon,\forall x,y\in\mathbb R.$$
Chứng minh rằng: $\exists$ hàm $g:\mathbb R\mapsto \mathbb R$ cộng tính sao cho $\left|f(x)-g(x)\right|\le \varepsilon$

$\bullet$ Cố định $t$, chứng minh tồn tại $\lim_{n\to \infty}\frac{f(3^nt)}{3^n}$.

Xây dựng hàm $g$ như sau

\[\begin{array}{rccl}g\colon &\mathbb{R}&\to &\mathbb{R}\\& t & \mapsto& \displaystyle\lim_{n\to \infty}\frac{f(3^nt)}{3^n}\end{array}\]

$\bullet$ Chứng minh $g$ cộng tính.

 

$\bullet$ Chứng minh $|f(x)-g(x)|\le \varepsilon$.




#733874 Trại hè hùng vương 2019

Đã gửi bởi nhungvienkimcuong on 03-07-2022 - 10:37 trong Thi HSG cấp Tỉnh, Thành phố. Olympic 30-4. Đề thi và kiểm tra đội tuyển các cấp.

Bài 5: (Đếm bằng hai cách cho "cặp cạnh cùng màu")

Ở đây ta quan tâm đến các cặp cạnh cùng màu và cùng xuất phát từ một điểm, ta sẽ gọi các cặp cạnh như vậy là "cặp cạnh đẹp". Dễ thấy tam giác có ba cạnh cùng màu thì sẽ có 3 cặp cạnh đẹp, và tam gíac có ba cạnh không cùng màu thì có 1 cặp cạnh đẹp.

Capture.PNG

Gọi $x$ là số tam giác có ba cạnh cùng màu, khi đó còn lại $20-x$ tam giác có ba cạnh không cùng màu. Dẫn đến
\[\#\{\text{cặp cạnh đẹp}\}=3x+(20-x)=2x+20.\tag{1}\]
Tại điểm $A$, gọi $a$ là số cạnh màu đỏ có $A$ là đầu mút, do đó có $5-a$ số cạnh màu xanh có $A$ là đầu mút. Số cặp cạnh đẹp thu được từ điểm $A$ là
\[\binom{a}{2}+\binom{5-a}{2}\ge \binom{2}{2}+\binom{3}{2}= 4.\]
Do vậy
\[\#\{\text{cặp cạnh đẹp}\}\ge \sum_{\text{điểm}\ A}4=24.\tag{2}\]
Từ $(1)$ và $(2)$ thu được $x\ge 2$, đây cũng chính là điều cần chứng minh.

 

Ghi chú: Sử dụng "cặp cạnh cùng màu" đã được viết trong một tập san mà Diễn đàn biên soạn năm 2006 (tải về tại đây, mục 8 - góc cùng màu).




#734125 Hàm tuần hoàn $f(x)=f(x+4)$

Đã gửi bởi nhungvienkimcuong on 28-07-2022 - 15:34 trong Phương trình hàm

Có vẻ như chưa thoả …

Cái $f^*$ em sai sót thật, nhưng cái $f^{**}$ thỏa đó thầy  :lol: . Sau đây em có chỉnh một tí so với ở trên

 

Với giả thiết đề cho thì ta thấy rằng chỉ cần quan tâm tới $x\in [0,4)$. Xét một hàm liên tục $g\colon [0,1]\to [0,1]$ bất kì thỏa mãn $g(0)=0,\ g(1)=1$ và với mọi $x\in (0,1)$ thì

$$g(x)\notin \{0,1\},\quad g(x)+g(1-x)=1.$$

Tiếp theo ta xét hàm $f_1\colon [0,2]\to [0,1]$ như sau

$$f_1(x)=\begin{cases} g(x) &, 0\le x\le 1\\ 1-g(x-1)&, 1<x\le 2\end{cases}$$

Khi đó hàm số $f^{**}\colon [0,4]\to [0,1]$ được xác định như sau

$$f^{**}(x)=\begin{cases}f_1(x)&, 0\le x\le 2\\ -f_1(x-2)&, 2<x\le 4 \end{cases}$$

thỏa mãn điều kiện đề cho.

 

Capture.PNG




#734122 Hàm tuần hoàn $f(x)=f(x+4)$

Đã gửi bởi nhungvienkimcuong on 28-07-2022 - 10:54 trong Phương trình hàm

Tìm tất cả các hàm số $f(x)$ liên tục, tuần hoàn, thoả mãn tất cả các điều kiện sau:
$$\left\{\begin{align*} f(x)&=f(x+4) \\ f(-x)&=-f(x) \\ \left|f(x)\right|+\left|f(x+1)\right| &=1,\,\, \forall x \\ f(x)&\ne 0,\,\,\forall x\ne 2n;\,\,n\in\mathbb Z\end{align*} \right.$$

Nếu thầy muốn thu được hàm số như trên thì vẫn chưa đủ giả thiết, vì ta vẫn có thể xây dựng hàm $f$ khả thoải mái với các điều kiện như này.

 

Đơn cử như sau, với giả thiết đề cho thì ta thấy rằng chỉ cần quan tâm với $x\in [0,4)$. Xét một hàm liên tục $g\colon [0,1]\to [-1,1]$ bất kì thỏa mãn $g(0)=0,g(1)\in\{-1,1\}$ và $g(x)\notin \{-1,0,1\}$ với mọi $x\in (0,1)$. Tiếp theo ta xét hàm $f_1\colon [0,2]\to [-1,1]$ như sau

$$f_1(x)=\begin{cases} g(x) &, 0\le x\le 1\\ g(1)-g(x-1)&, 1<x\le 2\end{cases}$$

Khi đó các hàm số $f^*,f^{**}\colon [0,4]\to [-1,1]$ được xác định như sau

$$f^*(x)=\begin{cases}f_1(x)&, 0\le x\le 2\\ f_1(x-2)&, 2<x\le 4 \end{cases},\quad f^{**}(x)=\begin{cases}f_1(x)&, 0\le x\le 2\\ -f_1(x-2)&, 2<x\le 4 \end{cases}$$

đều thỏa mãn điều kiện đề cho.




#734118 Hàm tuần hoàn $f(x)=f(x+4)$

Đã gửi bởi nhungvienkimcuong on 28-07-2022 - 08:00 trong Phương trình hàm

Tìm tất cả các hàm số $f(x)$ liên tục, tuần hoàn, thoả mãn tất cả các điều kiện sau:
$$\left\{\begin{align*} f(x)&=f(x+4) \\ f(-x)&=-f(x) \\ \left|f(x)\right|+\left|f(x+1)\right| &=1,\,\, \forall x\\ f(2n)&=0,\,\,\forall n\in\mathbb Z \end{align*} \right.$$

Em thấy rằng không cần giả thiết thứ tư, vì từ ba cái đầu sẽ suy ra được.

Thật vậy, từ điều kiện thứ hai suy ra $f(0)=0$. Do đó

$$|f(1)|=1\implies f(2)=0\implies |f(3)|=1\implies f(4)=0\implies \dots$$

Vậy ta có được $f(2n)=0$ với mọi $n\in \mathbb{N}$, thêm điều kiện hàm $f$ lẻ nên $f(2n)=0$ với mọi $n\in \mathbb{Z}$.